Difference between revisions of "2018 AMC 10B Problems/Problem 10"
Aops12142015 (talk | contribs) |
|||
Line 40: | Line 40: | ||
<math>\textbf{(A) } 1 \qquad \textbf{(B) } \frac{4}{3} \qquad \textbf{(C) } \frac{3}{2} \qquad \textbf{(D) } \frac{5}{3} \qquad \textbf{(E) } 2</math> | <math>\textbf{(A) } 1 \qquad \textbf{(B) } \frac{4}{3} \qquad \textbf{(C) } \frac{3}{2} \qquad \textbf{(D) } \frac{5}{3} \qquad \textbf{(E) } 2</math> | ||
+ | |||
+ | ==Solution== | ||
+ | Consider the cross-sectional plane. Note that <math>bh/2=3</math> and we want <math>bh/3</math>, so the answer is <math>2</math> |
Revision as of 14:54, 16 February 2018
In the rectangular parallelpiped shown, = , = , and = . Point is the midpoint of . What is the volume of the rectangular pyramid with base and apex ?
Solution
Consider the cross-sectional plane. Note that and we want , so the answer is